Kann sich durch Lorentz-Kontraktion ein Schwarzes Loch bilden? [Duplikat]

Mögliches Duplikat:
Wenn eine Masse von 1 kg auf nahezu Lichtgeschwindigkeit beschleunigt würde, würde sie sich in ein Schwarzes Loch verwandeln?

Stellen Sie sich vor, ein Stab der Länge L bewegt sich mit einer Geschwindigkeit, die sich der Lichtgeschwindigkeit in Bezug auf einen menschlichen Beobachter auf der Erde annähert. Aufgrund der Lorentz-Kontraktion wird beobachtet, dass der Stab sehr kurz ist. Und da alle Gesetze der Physik in jedem Bezugsrahmen gelten, wirkt das Gravitationsgesetz, das besagt, dass die wirkende Kraft umgekehrt proportional zum Quadrat des Abstands zwischen ihnen ist, zwischen verschiedenen Teilen der Stange. Wenn sich nun die Geschwindigkeit c nähert , nähert sich L 0. Dies sollte eine enorme Gravitationskraft verursachen, die ausreicht, um ein Schwarzes Loch zu bilden. Deutet dies nicht darauf hin, dass ein Schwarzes Loch entstehen kann, wenn die Objektgeschwindigkeit in die Nähe der Lichtgeschwindigkeit kommt?

Gemäß dem Bezugsrahmen des Stabs sieht er sich selbst als stationär an und ein Mensch, der ihn beobachtet, bewegt sich, also muss der Mensch gemäß dem Stab ein Schwarzes Loch sein. Ist das nicht ein Paradoxon?

Nicht wirklich ein Duplikat. Diese Frage erwähnt nicht die "Massenzunahme", sondern nur die Längenkontraktion.
@JohnRennie: Aber die Antworten auf die erste beantworten die zweite vollständig, dies ist ein Dup, Sie möchten nicht, dass die Leute den Text wörtlich wiederholen.
Ich glaube nicht, dass die Antworten auf die vorherige Frage die Frage tatsächlich beantworten. Sie alle sagen im Grunde nur "es würde dem Relativitätsprinzip widersprechen", und das ist keine Antwort. Ich war versucht zu sagen: "Das liegt daran, dass der Riemann-Tensor koordinateninvariant ist", aber das ist auch eine glatte Antwort. Was ich gerne sehen würde, sind einige Berechnungen, die mir zeigen, warum sich kein Schwarzes Loch bilden kann (offensichtlich kann es nicht!). Ich habe bisher nur eine fünfminütige Teepause darüber nachgedacht, aber ohne nennenswerte Fortschritte - vielleicht reicht meine Mittagspause :-)
Grundsätzlich besteht das Problem darin, dass Sie versuchen, die Newtonsche Gravitation, die spezielle Relativitätstheorie und die allgemeine Relativitätstheorie gleichzeitig anzuwenden. Das Paradoxon besagt im Grunde, dass Ihre Annahme, dass alle drei gleichzeitig auf dasselbe System angewendet werden können, dumm ist.
Nun, vielen Dank, Sie haben die Frage sechs Minuten, nachdem ich mich sehr bemüht hatte, sie zu beantworten, geschlossen!
Übrigens--@John, wenn Sie nicht genug Interesse wecken können, um diese Frage erneut zu öffnen, können Sie sie kennzeichnen, damit sie für die Zusammenführung mit dem möglichen Duplikat in Betracht gezogen wird. Oder Sie können Ihre Antwort einfach hier hinterlassen: Die Frage wird immer noch in der Suche angezeigt, die Leute können immer noch darüber abstimmen, und das OP kann Ihre Antwort immer noch akzeptieren, wenn er damit zufrieden ist. Ich nehme an, dass dies die bessere Version der Frage ist, aber dass es sich um Duplikate handelt.
Im Allgemeinen (dh nicht nur diese Frage) werden OP (und potenzielle Antwortende) im Geiste der SE-Vorschriften zur Vermeidung von Duplikaten ermutigt, vor dem Posten zu versuchen, Duplikate durch Site-Suchen zu erkennen, vgl. die FAQ. Wenn eine neue Frage ein Duplikat ist, erscheint es am logischsten, wenn neue Antworten an den ursprünglichen Beitrag gestellt werden, um das neue Duplikat nicht unnötig zu fördern.

Antworten (1)

OK, nachdem ich meine Mittagspause diesem Thema gewidmet habe (den Opfern, die ich für die Physik bringe!), habe ich eine Antwort für Sie. Ich bin mir nicht sicher, ob dies die bestmögliche Antwort ist. Wenn also jemand etwas verbessern kann, springen Sie bitte ein.

Erstens haben Sie absolut Recht zu sagen, dass die Dichte Ihres Objekts zunimmt, wenn es sich von Lorentz zusammenzieht. Dies ist keine Illusion: Das RHIC beobachtet dies jeden Tag. Beachten Sie, wie die Abbildungen auf der RHIC-Seite, die ich verlinkt habe, die kollidierenden Kerne zeigen, die zu Scheiben abgeflacht sind. Das kontrahierte Objekt kann jedoch kein Schwarzes Loch bilden, da dies gegen eines der Relativitätsprinzipien verstößt, dh das Vorhandensein oder Nichtvorhandensein des Schwarzen Lochs könnte verwendet werden, um festzustellen, wer sich bewegt und wer stillsteht. Was ist denn los?

Das Paradoxon ergibt sich aus Ihrer Annahme, dass die Masse / Dichte des Objekts bestimmt, ob es sich um ein Schwarzes Loch handelt oder nicht, da dies nicht wahr ist oder eher nur in Sonderfällen. Die Einstein-Gleichung , die uns die Krümmung und damit die Frage gibt, ob sich ein Schwarzes Loch bilden wird, lautet:

G a β = 8 π T a β

G a β ist der Einstein-Tensor , der die Krümmung beschreibt, während T a β ist der Spannungs-Energie-Tensor . Es ist also nicht die Masse/Dichte des Objekts, die die Krümmung bestimmt, sondern der Spannungs-Energie-Tensor.

Hier gibt es eine Abkürzung, denn der Spannungs-Energie-Tensor ist invariant, dh er ist in allen Koordinatensystemen gleich. Das bedeutet, dass der von uns beobachtete Spannungs-Energie-Tensor derselbe ist wie der im Ruhesystem Ihres Testobjekts beobachtete Spannungs-Energie-Tensor. Wenn also das Testobjekt in seinem Ruhesystem kein Schwarzes Loch bildet, bildet es in keinem anderen Koordinatensystem ein Schwarzes Loch , auch nicht in dem von Ihnen beschriebenen, in dem sich das Objekt fast mit Lichtgeschwindigkeit bewegt.

An diesem Punkt geht mir jedoch etwas die Luft aus, weshalb ich denke, dass diese Antwort verbessert werden kann. Es wäre schön, ein intuitives Gefühl dafür zu vermitteln, was der Stress-Energie-Tensor ist und warum er sich nicht ändert, wenn wir sehen, wie sich das Objekt fast mit Lichtgeschwindigkeit bewegt. Normalerweise schreiben wir den Stress-Energie-Tensor als 4 x 4-Matrix, und mit ein paar Annäherungen an Ihr Testobjekt hat der Tensor nur einen Wert ungleich Null, T 00 , was in der Tat die Dichte ist. Wenn wir den Stress-Energie-Tensor in unseren Rahmen schreiben, wo sich das Objekt bewegt, ist unser Wert für T 00 wird mit zunehmender Dichte zunehmen, und wenn sich sonst nichts ändern würde, würde dies schließlich ein Schwarzes Loch bilden. In unserem Rahmen sind die anderen Einträge in der Matrix jedoch nicht mehr Null. Die Änderungen in den anderen Einträgen gleichen die Änderung in der Dichte aus, sodass wir, wenn wir unseren Spannungs-Energie-Tensor in die Einstein-Gleichung einsetzen, die gleiche Krümmung wie im Ruhesystem des Testobjekts erhalten. Kein schwarzes Loch!

Das intuitive Zeug: Die Zeit-Raum-Komponenten sind der Impuls, und in der Grenze, die Sie diskutieren, sind sie gleich groß wie die Zeit-Zeit-Komponente. Zwei parallele Lichtbündel (oder zwei stark in die gleiche Richtung verstärkte Massen) ziehen sich nicht an – sie stoßen sich durch gravitomagnetische Kraft ab, um die Anziehung auszugleichen. Dies kann aus SR verstanden werden (denn wenn Sie zwei anziehende Objekte senkrecht zur Anziehungslinie verstärken, führt die Zeitdilatation dazu, dass die Zeit bis zur Kollision unendlich wird). Dasselbe gilt für EM, wo sich zwei auf nahe c verstärkte stationäre Ladungen nicht abstoßen (E und B heben sich auf).
Wenn sich ein Boot in Bewegung setzt, würde es aufgrund der Zunahme der relativistischen Energie sinken? Weil sich in diesem Fall die Stressenergie nicht ändern würde?
Um die Antwort zu ergänzen ... Im Spezialfall einer Radiuskugel R eher als eine Stange der Länge L . Ein Schwarzes Loch entsteht, wenn eine Masse vollständig in ihrem Schwarzschild-Radius eingeschlossen ist R S . Da die Kugel im Ruhezustand kein Schwarzes Loch ist: R > R S . Wenn die Kugel beschleunigt wird, schrumpft eine ihrer Richtungen auf beliebig kleine Längenskalen, aber die beiden Richtungen senkrecht zur Bewegung bleiben unverändert, und selbst im Grenzbereich einer flachen Scheibe befindet sich das Objekt immer noch außerhalb seines Schwarzschild-Radius: kein schwarzes Loch. @WizardOfMenlo
+1 Für deine verlorene Mittagspause. Diese Antwort ist besser als die in der doppelten Frage.
Nachdem Sie mehrere Dutzend Antworten auf diese Frage gelesen haben, ist dies bei weitem die beste. Insbesondere die Invarianz des Spannungsenergietensors sagt wirklich alles. Danke 10 Jahre später.